All about flooble | fun stuff | Get a free chatterbox | Free JavaScript | Avatars    
perplexus dot info

Home > Numbers
Block Distribution (Posted on 2024-04-23) Difficulty: 3 of 5
Ms Math's kindergarten class has 16 registered students. The classroom has a very large number, N, of play blocks which satisfies the conditions:

(a) If 16, 15, or 14 students are present in the class, then in each case all the blocks can be distributed in equal numbers to each student, and

(b) There are three integers 0 < x < y < z < 14 such that when x, y, or z students are present and the blocks are distributed in equal numbers to each student, there are exactly three blocks left over.

Find the sum of the distinct prime divisors of the least possible value of N satisfying the above conditions.

  Submitted by K Sengupta    
Rating: 5.0000 (1 votes)
Solution: (Hide)
Note that the number of play blocks is a multiple of the LCM of 16, 15, and 14. The value of this can be found to be (16)(15)(7) = 1680. This number is also divisible by 1, 2, 3, 4, 5, 6, 7, 8, 10, and 12, thus, the three numbers x, y, z are 9, 11, 13.

Thus, 1680k ≡3 when taken mod 9, 11, 13. Since 1680 is congruent to 6 mod 9 and 3 mod 13, and congruent to 8 mod 11, the number k must be a number that is congruent to 1 mod 13, 2 mod 3 (because 6 is a multiple of 3, which is a factor of 9 that can be divided out) and cause 8 to become 3 when multiplied under modulo 11.

Looking at the last condition shows that k≡ 10 mod 11 (after a bit of bashing) and is congruent to 1 mod 13 and 2 mod 3 as previously noted. Listing out the numbers congruent to 10 mod 11 and 1 mod 1 yield the following lists:
10 mod 11: 21, 32, 43, 54, 65, 76, 87, 98, 109, 120, 131...
1 mod 13: 14, 27, 40, 53, 66, 79, 92, 105, 118, 131, 144, 157, 170...

Both lists contain x elements where x is the modulo being taken, thus, there must be a solution in these lists as adding 11(13) to this solution yields the next smallest solution. In this case, 131 is the solution for $k$ and thus the answer is 1680(131). Since 131 is prime, the sum of the prime factors is $2 + 3 + 5 + 7 + 131 = 148.

Comments: ( You must be logged in to post comments.)
  Subject Author Date
re(2): Computer solutionLarry2024-04-24 10:44:19
re: Computer solutionCharlie2024-04-23 23:09:25
SolutionComputer solutionLarry2024-04-23 21:35:41
Solutioncomputer-aided solutionCharlie2024-04-23 14:49:10
Please log in:
Login:
Password:
Remember me:
Sign up! | Forgot password


Search:
Search body:
Forums (2)
Newest Problems
Random Problem
FAQ | About This Site
Site Statistics
New Comments (9)
Unsolved Problems
Top Rated Problems
This month's top
Most Commented On

Chatterbox:
Copyright © 2002 - 2024 by Animus Pactum Consulting. All rights reserved. Privacy Information